Last visit was: 14 May 2025, 23:55 It is currently 14 May 2025, 23:55
Close
GMAT Club Daily Prep
Thank you for using the timer - this advanced tool can estimate your performance and suggest more practice questions. We have subscribed you to Daily Prep Questions via email.

Customized
for You

we will pick new questions that match your level based on your Timer History

Track
Your Progress

every week, we’ll send you an estimated GMAT score based on your performance

Practice
Pays

we will pick new questions that match your level based on your Timer History
Not interested in getting valuable practice questions and articles delivered to your email? No problem, unsubscribe here.
Close
Request Expert Reply
Confirm Cancel
User avatar
agnok
Joined: 17 Nov 2009
Last visit: 08 May 2011
Posts: 202
Own Kudos:
304
 [26]
Given Kudos: 17
Posts: 202
Kudos: 304
 [26]
7
Kudos
Add Kudos
19
Bookmarks
Bookmark this Post
Most Helpful Reply
User avatar
Bunuel
User avatar
Math Expert
Joined: 02 Sep 2009
Last visit: 14 May 2025
Posts: 101,415
Own Kudos:
724,285
 [7]
Given Kudos: 93,515
Products:
Expert
Expert reply
Active GMAT Club Expert! Tag them with @ followed by their username for a faster response.
Posts: 101,415
Kudos: 724,285
 [7]
4
Kudos
Add Kudos
3
Bookmarks
Bookmark this Post
User avatar
KarishmaB
Joined: 16 Oct 2010
Last visit: 14 May 2025
Posts: 15,968
Own Kudos:
73,113
 [6]
Given Kudos: 467
Location: Pune, India
Expert
Expert reply
Active GMAT Club Expert! Tag them with @ followed by their username for a faster response.
Posts: 15,968
Kudos: 73,113
 [6]
3
Kudos
Add Kudos
3
Bookmarks
Bookmark this Post
General Discussion
avatar
DagwoodDeluxe
Joined: 12 Oct 2011
Last visit: 26 Aug 2016
Posts: 36
Own Kudos:
53
 [2]
Schools: HBS '15 (M)
GMAT 1: 760 Q49 V45
GPA: 3.94
Schools: HBS '15 (M)
GMAT 1: 760 Q49 V45
Posts: 36
Kudos: 53
 [2]
2
Kudos
Add Kudos
Bookmarks
Bookmark this Post
First, you need to simplify the stem. Since there is an absolute value in the expression, remove the absolute value signs and setup two inequalities.

A. x-3 < 7 ?
x < 10 ?

B. -(x-3) < 7 ?
-x+3 < 7 ?
-x < 4 ?
x > 4 ?

Next, combine the two inequalities: -4 < x < 10 ? In other words, the question is asking whether x is between -4 and 10.

Now, evaluate the statements.

(1) x > 0
INSUFFICIENT: because x>0, x>-4. However, the statement does not prove that x<10. For example, if x = 8, the answer to the question is yes. If x = 15, the answer to the question is no.

(2) x < 10
INSUFFICIENT: this statment indicates that x<10. However, the statement does not prove that x>-4. For example, if x = 8, the answer to the question is yes. If x = -10, the answer to the question is no.

Finally, combine the two statements. Combined, the two statements say: 0<x<10. If x is between 0 and 10, x must also be between -4 and 10. Therefore, the answer to the question is C.

Answer: C
User avatar
Bunuel
User avatar
Math Expert
Joined: 02 Sep 2009
Last visit: 14 May 2025
Posts: 101,415
Own Kudos:
724,285
 [4]
Given Kudos: 93,515
Products:
Expert
Expert reply
Active GMAT Club Expert! Tag them with @ followed by their username for a faster response.
Posts: 101,415
Kudos: 724,285
 [4]
3
Kudos
Add Kudos
1
Bookmarks
Bookmark this Post
One can also use distance concept to solve this questions, which for this particular question would be an easier approach.

Is |x-3| < 7 ?

|x-3| is just the distance between 3 and x on the number line. The question basically asks whether this distance is less than 7: --(-4)------(3)------(10)-- so, whether -4<x<10 is true?

(1) x > 0. Not sufficient.
(2) x < 10. Not sufficient.

(1)+(2) Intersection of the ranges from (1) and (2) is 0<x<10, so the answer to the question is YES. Sufficient.

Answer: C.
User avatar
Apex231
Joined: 03 Oct 2009
Last visit: 14 Jun 2014
Posts: 37
Own Kudos:
Given Kudos: 8
Schools: ISB '14
Posts: 37
Kudos: 667
Kudos
Add Kudos
Bookmarks
Bookmark this Post
DagwoodDeluxe
First, you need to simplify the stem. Since there is an absolute value in the expression, remove the absolute value signs and setup two inequalities.

A. x-3 < 7 ?
x < 10 ?

B. -(x-3) < 7 ?
-x+3 < 7 ?
-x < 4 ?
x > 4 ?

Next, combine the two inequalities: -4 < x < 10 ? In other words, the question is asking whether x is between -4 and 10.

Is |x – 3| < 7 ?

x-3 < 7
x < 10
OR
-x+3 < 7
x > -4


When we consider both inequalities, question becomes -

is x < 10
OR
is x > -4

Isn't it wrong to combine inequalities like this -4 < x < 10 and consider two scenarios as AND scenarios?
User avatar
Bunuel
User avatar
Math Expert
Joined: 02 Sep 2009
Last visit: 14 May 2025
Posts: 101,415
Own Kudos:
Given Kudos: 93,515
Products:
Expert
Expert reply
Active GMAT Club Expert! Tag them with @ followed by their username for a faster response.
Posts: 101,415
Kudos: 724,285
Kudos
Add Kudos
Bookmarks
Bookmark this Post
Apex231
Is |x – 3| < 7 ?

x-3 < 7
x < 10
OR
-x+3 < 7
x > -4


When we consider both inequalities, question becomes -

is x < 10
OR
is x > -4

Isn't it wrong to combine inequalities like this -4 < x < 10 and consider two scenarios as AND scenarios?

"Is |x-3|<7?" does mean "is -4<x<10?" Refer to my solution above.

Or another way: if x<3, then -(x-3)<7 --> x>-4 AND if x>=3, then x-3<7 --> x<10. So, |x-3|<7 to holds true for -4<x<10. So, we don't have 2 separate inequalities: x>-4 OR x<10, we have 2 cases and both must hold true for |x-3|<7 to hold true.

Hope it's clear.
User avatar
Apex231
Joined: 03 Oct 2009
Last visit: 14 Jun 2014
Posts: 37
Own Kudos:
Given Kudos: 8
Schools: ISB '14
Posts: 37
Kudos: 667
Kudos
Add Kudos
Bookmarks
Bookmark this Post
In the post below we considered two scenarios as OR scenarios and in this question we are considering them as AND scenarios. I understand the distance approach but still little confused. How is this question different from one below? Please help!

https://gmatclub.com/forum/ds-inequalities-126369.html

In the question below we considered two cases separately and didn't combine i.e. didn't do y + 3 <= x <= 3 - y

If y >= 0, What is the value of x?
(1) |x-3| >= y
(2) |x-3| <= -y

1) |x-3| >= y

x -3 >=y
x >= y + 3

OR

3 - x >=y
x <= 3 - y

Posted from my mobile device
User avatar
Bunuel
User avatar
Math Expert
Joined: 02 Sep 2009
Last visit: 14 May 2025
Posts: 101,415
Own Kudos:
724,285
 [1]
Given Kudos: 93,515
Products:
Expert
Expert reply
Active GMAT Club Expert! Tag them with @ followed by their username for a faster response.
Posts: 101,415
Kudos: 724,285
 [1]
1
Kudos
Add Kudos
Bookmarks
Bookmark this Post
Apex231
In the post below we considered two scenarios as OR scenarios and in this question we are considering them as AND scenarios. I understand the distance approach but still little confused. How is this question different from one below? Please help!

Forget about or, and and y. The point was that you were combining two cases into one when you couldn't do that.

Consider this: |x-3|>1
If x<=3 --> -(x-3)>1 --> x<2;
If x>3 --> (x-3)>1 --> x>4.

And then you were writing 4<x<2, which is obviously wrong.

Another example: |x-3|<1
If x<=3 --> -(x-3)<1 --> 2<x --> 2<x<=3;
If x>3 --> (x-3)>1 --> x<4. --> 3<x<4;
Now you can write 2<x<4 as it would be right.

Hope it's clear.
User avatar
Apex231
Joined: 03 Oct 2009
Last visit: 14 Jun 2014
Posts: 37
Own Kudos:
Given Kudos: 8
Schools: ISB '14
Posts: 37
Kudos: 667
Kudos
Add Kudos
Bookmarks
Bookmark this Post
Thanks Bunuel. I think i understand now.

So in the first example we can combine ranges because it's a continuous range but in the second example the range is not continuous so we can't combine.
User avatar
daboo343
Joined: 09 Nov 2012
Last visit: 20 Nov 2022
Posts: 223
Own Kudos:
686
 [1]
Given Kudos: 162
Status:You have to have the darkness for the dawn to come
Daboo: Sonu
GMAT 1: 590 Q49 V20
GMAT 2: 730 Q50 V38
Products:
GMAT 2: 730 Q50 V38
Posts: 223
Kudos: 686
 [1]
1
Kudos
Add Kudos
Bookmarks
Bookmark this Post
DesecratoR
Is |x – 3| < 7 ?

(1) x > 0
(2) x < 10

Need explanation please. Thank you!

we have to find the value of lx-3l<7
simplify this statement
-7<(x-3)<7
on solving this equation we get
-4<x<10
so basically we have to find whether the of x lies betwee -4 and 10

Statement 1
x>0
so from this we can say the value of x is greater than 0 and can be any positive value fro example 50
so A is not possible
Statement 2
x<10
similarly from this we only know value of x< 10 and can even -50
so B is also not possible

on combining both statement we can conclude 0<x<10
and after combining we get that the vaue of x is in between -4<x<10
So C is ans
User avatar
daboo343
Joined: 09 Nov 2012
Last visit: 20 Nov 2022
Posts: 223
Own Kudos:
686
 [1]
Given Kudos: 162
Status:You have to have the darkness for the dawn to come
Daboo: Sonu
GMAT 1: 590 Q49 V20
GMAT 2: 730 Q50 V38
Products:
GMAT 2: 730 Q50 V38
Posts: 223
Kudos: 686
 [1]
Kudos
Add Kudos
1
Bookmarks
Bookmark this Post
DesecratoR
Is |x – 3| < 7 ?

(1) x > 0
(2) x < 10

Need explanation please. Thank you!

we have to find the value of lx-3l<7
simplify this statement
-7<(x-3)<7
on solving this equation we get
-4<x<10
so basically we have to find whether the of x lies betwee -4 and 10

Statement 1
x>0
so from this we can say the value of x is greater than 0 and can be any positive value fro example 50
so A is not possible
Statement 2
x<10
similarly from this we only know value of x< 10 and can even -50
so B is also not possible

on combining both statement we can conclude 0<x<10
and after combining we get that the vaue of x is in between -4<x<10
So C is ans
Kudos
Add Kudos
Bookmarks
Bookmark this Post
agnok
Is |x – 3| < 7 ?

(1) x > 0
(2) x < 10
The question stem says:
Is -4<x<10?
In number line, it is green part.
<----(-5)----(-4)---(-3)---(-2)---(-1)---0----1---2---3---4---5---6---7---8---9---10----11>
Statement 1:
x>0
here, x may be any positive values. This may be in the green part or may be next after the green part.
So, insufficient.
Statement 2:
x<10
Here, x may be into the green part or it may be in the red part. So, the answer may be YES or NO. So, insufficient.
Statement 1+2:
It combined says:
0<x<10
So, sufficient from the number line. The correct choice is C.
avatar
sampad
Joined: 13 Jun 2019
Last visit: 02 Mar 2022
Posts: 38
Own Kudos:
Given Kudos: 5
Posts: 38
Kudos: 9
Kudos
Add Kudos
Bookmarks
Bookmark this Post
from the question stem we need to find -4<x<10. using both the statement we get 0<x<10. Although it is the subset of the question stem. But we are not getting the exact answer. So it should be insufficient. there is no other constraints mentioned whether x in positive integer or anything. Not understand why it is C.
User avatar
Bunuel
User avatar
Math Expert
Joined: 02 Sep 2009
Last visit: 14 May 2025
Posts: 101,415
Own Kudos:
Given Kudos: 93,515
Products:
Expert
Expert reply
Active GMAT Club Expert! Tag them with @ followed by their username for a faster response.
Posts: 101,415
Kudos: 724,285
Kudos
Add Kudos
Bookmarks
Bookmark this Post
sampad
from the question stem we need to find -4<x<10. using both the statement we get 0<x<10. Although it is the subset of the question stem. But we are not getting the exact answer. So it should be insufficient. there is no other constraints mentioned whether x in positive integer or anything. Not understand why it is C.

This is an YES/NO DS question. In a Yes/No Data Sufficiency questions, statement(s) is sufficient if the answer is “always yes” or “always no” while a statement(s) is insufficient if the answer is "sometimes yes" and "sometimes no".

The above question asks: is -4 < x < 10 ? When combining the statements we get that 0 < x < 10. Now, does this allow us to answer the question with definite YES or definite NO? If 0 < x < 10, then x is for sure between -4 and 10, so the answer to the question whether -4 < x < 10 is YES.

Hope it's clear.
User avatar
Archit3110
User avatar
Major Poster
Joined: 18 Aug 2017
Last visit: 14 May 2025
Posts: 8,209
Own Kudos:
Given Kudos: 243
Status:You learn more from failure than from success.
Location: India
Concentration: Sustainability, Marketing
GMAT Focus 1: 545 Q79 V79 DI73
GPA: 4
WE:Marketing (Energy)
Products:
GMAT Focus 1: 545 Q79 V79 DI73
Posts: 8,209
Kudos: 4,705
Kudos
Add Kudos
Bookmarks
Bookmark this Post
agnok
Is |x – 3| < 7 ?

(1) x > 0
(2) x < 10

the given inequality |x – 3| < 7 can be written as
-4<x<10
#1
x>0 insufficient
#2
x<10 again insufficient
from 1 &2
sufficient
10>x<0
IMOC
User avatar
Basshead
Joined: 09 Jan 2020
Last visit: 07 Feb 2024
Posts: 929
Own Kudos:
Given Kudos: 432
Location: United States
Posts: 929
Kudos: 276
Kudos
Add Kudos
Bookmarks
Bookmark this Post
Is |x – 3| < 7 ?
Is -4 < x < 10?

(1) x > 0

x could be 11. Insufficient.

(2) x < 10

x could be -5. Insufficient.

(1&2) 0 < x < 10

This is within our range. Sufficient.

Answer is C.
User avatar
jhavyom
Joined: 02 Sep 2019
Last visit: 17 Dec 2022
Posts: 174
Own Kudos:
Given Kudos: 28
Location: India
Schools: ISB'22
Schools: ISB'22
Posts: 174
Kudos: 264
Kudos
Add Kudos
Bookmarks
Bookmark this Post
Is |x – 3| > 7?

|x – 3| > 7
x - 3 > 7 or x - 3 < -7
x > 10 or x < -4

(1) x > 10
Sufficient

(2) x > –10
Not sufficient

Choice A is the answer.
User avatar
bumpbot
User avatar
Non-Human User
Joined: 09 Sep 2013
Last visit: 04 Jan 2021
Posts: 36,858
Own Kudos:
Posts: 36,858
Kudos: 983
Kudos
Add Kudos
Bookmarks
Bookmark this Post
Hello from the GMAT Club BumpBot!

Thanks to another GMAT Club member, I have just discovered this valuable topic, yet it had no discussion for over a year. I am now bumping it up - doing my job. I think you may find it valuable (esp those replies with Kudos).

Want to see all other topics I dig out? Follow me (click follow button on profile). You will receive a summary of all topics I bump in your profile area as well as via email.
Moderator:
Math Expert
101415 posts